An example of an infinite open cover of the interval (0,1) that has no finite subcoverWhy can't a open...

Should my PhD thesis be submitted under my legal name?

How will losing mobility of one hand affect my career as a programmer?

Who must act to prevent Brexit on March 29th?

Can the electrostatic force be infinite in magnitude?

Can I use my Chinese passport to enter China after I acquired another citizenship?

My boss asked me to take a one-day class, then signs it up as a day off

Why is delta-v is the most useful quantity for planning space travel?

Simple recursive Sudoku solver

Adding empty element to declared container without declaring type of element

Identify a stage play about a VR experience in which participants are encouraged to simulate performing horrific activities

Installing PowerShell on 32-bit Kali OS fails

Giant Toughroad SLR 2 for 200 miles in two days, will it make it?

For airliners, what prevents wing strikes on landing in bad weather?

Greatest common substring

What (else) happened July 1st 1858 in London?

Is there enough fresh water in the world to eradicate the drinking water crisis?

How do ultrasonic sensors differentiate between transmitted and received signals?

Science Fiction story where a man invents a machine that can help him watch history unfold

In Star Trek IV, why did the Bounty go back to a time when whales were already rare?

What will be the benefits of Brexit?

Superhero words!

Golf game boilerplate

Java - What do constructor type arguments mean when placed *before* the type?

How to deal with or prevent idle in the test team?



An example of an infinite open cover of the interval (0,1) that has no finite subcover


Why can't a open interval in $mathbb{R}$ be compact?Finite sub cover for $(0,1)$For which compact sets can the size of the finite subcover be bounded?Open Cover of Compact Set Minus a Point on the Boundaryopen cover and finite subcoverExample of open cover of (0,1) which has no finite subcoverUsing the “open cover” definition of compactness to show that $[0,1]$ is compactFind an open cover of the interval $(-1,1)$ that has no finite subcoverDoes there exist an infinite number of open intervals that cover $[0,1]$ but for a finite number of intervals it does not cover $[0,1]$?Every open cover of $[0,1]$ has finite subcoverCompactness of the closed interval [0,1]













7












$begingroup$


I've been having a hard time solving this problem that I was given in class. The problem states
" Give an example of an infinite open cover of the interval (0,1) that has no finite subcover."



I know that the set has to be compact and that both 0 and 1 are limit points. Aside from those two known factors I'm at a complete loss as to how to go about solving this problem.










share|cite|improve this question











$endgroup$








  • 1




    $begingroup$
    What set has to be compact?
    $endgroup$
    – user64687
    Feb 4 '15 at 17:06
















7












$begingroup$


I've been having a hard time solving this problem that I was given in class. The problem states
" Give an example of an infinite open cover of the interval (0,1) that has no finite subcover."



I know that the set has to be compact and that both 0 and 1 are limit points. Aside from those two known factors I'm at a complete loss as to how to go about solving this problem.










share|cite|improve this question











$endgroup$








  • 1




    $begingroup$
    What set has to be compact?
    $endgroup$
    – user64687
    Feb 4 '15 at 17:06














7












7








7


3



$begingroup$


I've been having a hard time solving this problem that I was given in class. The problem states
" Give an example of an infinite open cover of the interval (0,1) that has no finite subcover."



I know that the set has to be compact and that both 0 and 1 are limit points. Aside from those two known factors I'm at a complete loss as to how to go about solving this problem.










share|cite|improve this question











$endgroup$




I've been having a hard time solving this problem that I was given in class. The problem states
" Give an example of an infinite open cover of the interval (0,1) that has no finite subcover."



I know that the set has to be compact and that both 0 and 1 are limit points. Aside from those two known factors I'm at a complete loss as to how to go about solving this problem.







real-analysis general-topology compactness






share|cite|improve this question















share|cite|improve this question













share|cite|improve this question




share|cite|improve this question








edited Feb 4 '15 at 18:13









Asaf Karagila

307k33439770




307k33439770










asked Feb 4 '15 at 17:03









Chris MillettChris Millett

1571113




1571113








  • 1




    $begingroup$
    What set has to be compact?
    $endgroup$
    – user64687
    Feb 4 '15 at 17:06














  • 1




    $begingroup$
    What set has to be compact?
    $endgroup$
    – user64687
    Feb 4 '15 at 17:06








1




1




$begingroup$
What set has to be compact?
$endgroup$
– user64687
Feb 4 '15 at 17:06




$begingroup$
What set has to be compact?
$endgroup$
– user64687
Feb 4 '15 at 17:06










5 Answers
5






active

oldest

votes


















13












$begingroup$

Actually, if you haven't already learned this, you will learn that in $Bbb R^{n}$, a set is compact if any only if it is both closed and bounded.



The set $(0,1)$ is bounded, but it is not closed, so it can't be compact.



The solution to your exercise of finding an open cover with no finite subcover proves that $(0,1)$ is not compact, because the definition of a set being compact is that every open cover of the set has a finite subcover.



So, the whole trick to finding an open cover with no finite subcover is this:



$(0,1)$ is not closed, and so it doesn't contain all of its limit points (it's easy to see that the two it doesn't contain are $0$ and $1$). Well, can you construct an open cover whose open sets get closer and closer and closer to at least one (or maybe both) of these end points, but never quite reaches them? Hint: since we are in $Bbb R$, think about how you would get closer and closer to a real number $r$ without ever actually being $r$. If you want to get closer from above, then ${r + frac{1}{n} }_{n = 1}^{infty}$ is a sequence that gets closer to $r$ from above. What would be a sequence that gets close to $r$ from below? You should hopefully say ${r - frac{1}{n} }_{n = 1}^{infty}$.



Anyway, so maybe we can construct our open intervals so that the end points get closer and closer to the limit points $0$ and $1$, but finitely many of the sets would always have gaps between them and $0$ and $1$... hmm...



Well, we could do $(0 + frac{1}{3}, 1 - frac{1}{3}) cup (0 + frac{1}{4}, 1 - frac{1}{4}) (0 + frac{1}{5}, 1 - frac{1}{5}) cup ...$.



These open sets are in $(0,1)$ and have end points getting closer and closer to $0$ and $1$, but any finite number of them won't contain all of $(0,1)$ (why?). If we use a formula to define these open intervals, it would be ${ (0 + frac{1}{n}, 1 - frac{1}{n}) }_{n = 3}^{infty}$.






share|cite|improve this answer











$endgroup$









  • 1




    $begingroup$
    Of the four answers so far, this is the only one that answers the question of “how to go about solving this problem”.
    $endgroup$
    – MJD
    Feb 4 '15 at 17:48










  • $begingroup$
    @user46944 thank you so much for explaining this. He said that this one would make us think for a while. I assume he said this because we haven't fully touched upon this yet. Thanks for the indepth and comprehensive answer
    $endgroup$
    – Chris Millett
    Feb 4 '15 at 18:59










  • $begingroup$
    @ChrisMillet You're welcome! I hope it makes sense.
    $endgroup$
    – layman
    Feb 4 '15 at 20:33





















2












$begingroup$

Think about...
$$
bigcup_{n=1}^{infty}left(0+frac{1}{n},1right)
$$






share|cite|improve this answer









$endgroup$





















    2












    $begingroup$

    You can take the open cover ${(0+frac{1}{n}, 1-frac{1}{n})mid n>2} $. It has no finite subcover because if it does simply take the largest $n$ and $frac{1}{2n}$ is not included.






    share|cite|improve this answer









    $endgroup$













    • $begingroup$
      I never thought of it like that. I assume it's because we havent fully finished the topic yet and I wasn't sure whether or not to keep reading thanks!
      $endgroup$
      – Chris Millett
      Feb 4 '15 at 19:03



















    2












    $begingroup$

    Since you have the real analysis tag, I'm assuming you are using the standard topology on $Bbb{R}$, so open sets look like intervals of the form $(a,b)$. An open cover of $(0,1)$ could look something like $$bigcup_{i=1}^infty left(0,1-frac{1}{i} right)$$ But you need to verify this is an open cover. That is, show $$(0,1) subseteq bigcup_{i=1}^infty left(0,1-frac{1}{i} right)$$ and further show that if $K subset Bbb{N}$ is any finite subset of $Bbb{N}$ that $$(0,1) notsubseteq bigcup_{iin K} left(0,1-frac{1}{i} right)$$ which demonstrates that the open cover has no finite subcover.






    share|cite|improve this answer









    $endgroup$













    • $begingroup$
      gotcha this makes sense too. Thanks a bunch
      $endgroup$
      – Chris Millett
      Feb 4 '15 at 19:00



















    0












    $begingroup$

    In case you are still interested. Abbot's has this problem as Example 3.3.7. For each point $xin(0,1)$ consider the open interval $0_x = (x/2,1)$. The infinite union of $O_x$'s forms an open cover of $(0,1)$. Take any finite subset of such $O_x$, and set $x'=operatorname{min}(x_1,x_2,dots,x_n)$. You can still find a number $0<yleq x'/2$ not in any subfinite union.






    share|cite|improve this answer









    $endgroup$













      Your Answer





      StackExchange.ifUsing("editor", function () {
      return StackExchange.using("mathjaxEditing", function () {
      StackExchange.MarkdownEditor.creationCallbacks.add(function (editor, postfix) {
      StackExchange.mathjaxEditing.prepareWmdForMathJax(editor, postfix, [["$", "$"], ["\\(","\\)"]]);
      });
      });
      }, "mathjax-editing");

      StackExchange.ready(function() {
      var channelOptions = {
      tags: "".split(" "),
      id: "69"
      };
      initTagRenderer("".split(" "), "".split(" "), channelOptions);

      StackExchange.using("externalEditor", function() {
      // Have to fire editor after snippets, if snippets enabled
      if (StackExchange.settings.snippets.snippetsEnabled) {
      StackExchange.using("snippets", function() {
      createEditor();
      });
      }
      else {
      createEditor();
      }
      });

      function createEditor() {
      StackExchange.prepareEditor({
      heartbeatType: 'answer',
      autoActivateHeartbeat: false,
      convertImagesToLinks: true,
      noModals: true,
      showLowRepImageUploadWarning: true,
      reputationToPostImages: 10,
      bindNavPrevention: true,
      postfix: "",
      imageUploader: {
      brandingHtml: "Powered by u003ca class="icon-imgur-white" href="https://imgur.com/"u003eu003c/au003e",
      contentPolicyHtml: "User contributions licensed under u003ca href="https://creativecommons.org/licenses/by-sa/3.0/"u003ecc by-sa 3.0 with attribution requiredu003c/au003e u003ca href="https://stackoverflow.com/legal/content-policy"u003e(content policy)u003c/au003e",
      allowUrls: true
      },
      noCode: true, onDemand: true,
      discardSelector: ".discard-answer"
      ,immediatelyShowMarkdownHelp:true
      });


      }
      });














      draft saved

      draft discarded


















      StackExchange.ready(
      function () {
      StackExchange.openid.initPostLogin('.new-post-login', 'https%3a%2f%2fmath.stackexchange.com%2fquestions%2f1133580%2fan-example-of-an-infinite-open-cover-of-the-interval-0-1-that-has-no-finite-su%23new-answer', 'question_page');
      }
      );

      Post as a guest















      Required, but never shown

























      5 Answers
      5






      active

      oldest

      votes








      5 Answers
      5






      active

      oldest

      votes









      active

      oldest

      votes






      active

      oldest

      votes









      13












      $begingroup$

      Actually, if you haven't already learned this, you will learn that in $Bbb R^{n}$, a set is compact if any only if it is both closed and bounded.



      The set $(0,1)$ is bounded, but it is not closed, so it can't be compact.



      The solution to your exercise of finding an open cover with no finite subcover proves that $(0,1)$ is not compact, because the definition of a set being compact is that every open cover of the set has a finite subcover.



      So, the whole trick to finding an open cover with no finite subcover is this:



      $(0,1)$ is not closed, and so it doesn't contain all of its limit points (it's easy to see that the two it doesn't contain are $0$ and $1$). Well, can you construct an open cover whose open sets get closer and closer and closer to at least one (or maybe both) of these end points, but never quite reaches them? Hint: since we are in $Bbb R$, think about how you would get closer and closer to a real number $r$ without ever actually being $r$. If you want to get closer from above, then ${r + frac{1}{n} }_{n = 1}^{infty}$ is a sequence that gets closer to $r$ from above. What would be a sequence that gets close to $r$ from below? You should hopefully say ${r - frac{1}{n} }_{n = 1}^{infty}$.



      Anyway, so maybe we can construct our open intervals so that the end points get closer and closer to the limit points $0$ and $1$, but finitely many of the sets would always have gaps between them and $0$ and $1$... hmm...



      Well, we could do $(0 + frac{1}{3}, 1 - frac{1}{3}) cup (0 + frac{1}{4}, 1 - frac{1}{4}) (0 + frac{1}{5}, 1 - frac{1}{5}) cup ...$.



      These open sets are in $(0,1)$ and have end points getting closer and closer to $0$ and $1$, but any finite number of them won't contain all of $(0,1)$ (why?). If we use a formula to define these open intervals, it would be ${ (0 + frac{1}{n}, 1 - frac{1}{n}) }_{n = 3}^{infty}$.






      share|cite|improve this answer











      $endgroup$









      • 1




        $begingroup$
        Of the four answers so far, this is the only one that answers the question of “how to go about solving this problem”.
        $endgroup$
        – MJD
        Feb 4 '15 at 17:48










      • $begingroup$
        @user46944 thank you so much for explaining this. He said that this one would make us think for a while. I assume he said this because we haven't fully touched upon this yet. Thanks for the indepth and comprehensive answer
        $endgroup$
        – Chris Millett
        Feb 4 '15 at 18:59










      • $begingroup$
        @ChrisMillet You're welcome! I hope it makes sense.
        $endgroup$
        – layman
        Feb 4 '15 at 20:33


















      13












      $begingroup$

      Actually, if you haven't already learned this, you will learn that in $Bbb R^{n}$, a set is compact if any only if it is both closed and bounded.



      The set $(0,1)$ is bounded, but it is not closed, so it can't be compact.



      The solution to your exercise of finding an open cover with no finite subcover proves that $(0,1)$ is not compact, because the definition of a set being compact is that every open cover of the set has a finite subcover.



      So, the whole trick to finding an open cover with no finite subcover is this:



      $(0,1)$ is not closed, and so it doesn't contain all of its limit points (it's easy to see that the two it doesn't contain are $0$ and $1$). Well, can you construct an open cover whose open sets get closer and closer and closer to at least one (or maybe both) of these end points, but never quite reaches them? Hint: since we are in $Bbb R$, think about how you would get closer and closer to a real number $r$ without ever actually being $r$. If you want to get closer from above, then ${r + frac{1}{n} }_{n = 1}^{infty}$ is a sequence that gets closer to $r$ from above. What would be a sequence that gets close to $r$ from below? You should hopefully say ${r - frac{1}{n} }_{n = 1}^{infty}$.



      Anyway, so maybe we can construct our open intervals so that the end points get closer and closer to the limit points $0$ and $1$, but finitely many of the sets would always have gaps between them and $0$ and $1$... hmm...



      Well, we could do $(0 + frac{1}{3}, 1 - frac{1}{3}) cup (0 + frac{1}{4}, 1 - frac{1}{4}) (0 + frac{1}{5}, 1 - frac{1}{5}) cup ...$.



      These open sets are in $(0,1)$ and have end points getting closer and closer to $0$ and $1$, but any finite number of them won't contain all of $(0,1)$ (why?). If we use a formula to define these open intervals, it would be ${ (0 + frac{1}{n}, 1 - frac{1}{n}) }_{n = 3}^{infty}$.






      share|cite|improve this answer











      $endgroup$









      • 1




        $begingroup$
        Of the four answers so far, this is the only one that answers the question of “how to go about solving this problem”.
        $endgroup$
        – MJD
        Feb 4 '15 at 17:48










      • $begingroup$
        @user46944 thank you so much for explaining this. He said that this one would make us think for a while. I assume he said this because we haven't fully touched upon this yet. Thanks for the indepth and comprehensive answer
        $endgroup$
        – Chris Millett
        Feb 4 '15 at 18:59










      • $begingroup$
        @ChrisMillet You're welcome! I hope it makes sense.
        $endgroup$
        – layman
        Feb 4 '15 at 20:33
















      13












      13








      13





      $begingroup$

      Actually, if you haven't already learned this, you will learn that in $Bbb R^{n}$, a set is compact if any only if it is both closed and bounded.



      The set $(0,1)$ is bounded, but it is not closed, so it can't be compact.



      The solution to your exercise of finding an open cover with no finite subcover proves that $(0,1)$ is not compact, because the definition of a set being compact is that every open cover of the set has a finite subcover.



      So, the whole trick to finding an open cover with no finite subcover is this:



      $(0,1)$ is not closed, and so it doesn't contain all of its limit points (it's easy to see that the two it doesn't contain are $0$ and $1$). Well, can you construct an open cover whose open sets get closer and closer and closer to at least one (or maybe both) of these end points, but never quite reaches them? Hint: since we are in $Bbb R$, think about how you would get closer and closer to a real number $r$ without ever actually being $r$. If you want to get closer from above, then ${r + frac{1}{n} }_{n = 1}^{infty}$ is a sequence that gets closer to $r$ from above. What would be a sequence that gets close to $r$ from below? You should hopefully say ${r - frac{1}{n} }_{n = 1}^{infty}$.



      Anyway, so maybe we can construct our open intervals so that the end points get closer and closer to the limit points $0$ and $1$, but finitely many of the sets would always have gaps between them and $0$ and $1$... hmm...



      Well, we could do $(0 + frac{1}{3}, 1 - frac{1}{3}) cup (0 + frac{1}{4}, 1 - frac{1}{4}) (0 + frac{1}{5}, 1 - frac{1}{5}) cup ...$.



      These open sets are in $(0,1)$ and have end points getting closer and closer to $0$ and $1$, but any finite number of them won't contain all of $(0,1)$ (why?). If we use a formula to define these open intervals, it would be ${ (0 + frac{1}{n}, 1 - frac{1}{n}) }_{n = 3}^{infty}$.






      share|cite|improve this answer











      $endgroup$



      Actually, if you haven't already learned this, you will learn that in $Bbb R^{n}$, a set is compact if any only if it is both closed and bounded.



      The set $(0,1)$ is bounded, but it is not closed, so it can't be compact.



      The solution to your exercise of finding an open cover with no finite subcover proves that $(0,1)$ is not compact, because the definition of a set being compact is that every open cover of the set has a finite subcover.



      So, the whole trick to finding an open cover with no finite subcover is this:



      $(0,1)$ is not closed, and so it doesn't contain all of its limit points (it's easy to see that the two it doesn't contain are $0$ and $1$). Well, can you construct an open cover whose open sets get closer and closer and closer to at least one (or maybe both) of these end points, but never quite reaches them? Hint: since we are in $Bbb R$, think about how you would get closer and closer to a real number $r$ without ever actually being $r$. If you want to get closer from above, then ${r + frac{1}{n} }_{n = 1}^{infty}$ is a sequence that gets closer to $r$ from above. What would be a sequence that gets close to $r$ from below? You should hopefully say ${r - frac{1}{n} }_{n = 1}^{infty}$.



      Anyway, so maybe we can construct our open intervals so that the end points get closer and closer to the limit points $0$ and $1$, but finitely many of the sets would always have gaps between them and $0$ and $1$... hmm...



      Well, we could do $(0 + frac{1}{3}, 1 - frac{1}{3}) cup (0 + frac{1}{4}, 1 - frac{1}{4}) (0 + frac{1}{5}, 1 - frac{1}{5}) cup ...$.



      These open sets are in $(0,1)$ and have end points getting closer and closer to $0$ and $1$, but any finite number of them won't contain all of $(0,1)$ (why?). If we use a formula to define these open intervals, it would be ${ (0 + frac{1}{n}, 1 - frac{1}{n}) }_{n = 3}^{infty}$.







      share|cite|improve this answer














      share|cite|improve this answer



      share|cite|improve this answer








      edited Feb 4 '15 at 18:04

























      answered Feb 4 '15 at 17:16









      laymanlayman

      15.2k22256




      15.2k22256








      • 1




        $begingroup$
        Of the four answers so far, this is the only one that answers the question of “how to go about solving this problem”.
        $endgroup$
        – MJD
        Feb 4 '15 at 17:48










      • $begingroup$
        @user46944 thank you so much for explaining this. He said that this one would make us think for a while. I assume he said this because we haven't fully touched upon this yet. Thanks for the indepth and comprehensive answer
        $endgroup$
        – Chris Millett
        Feb 4 '15 at 18:59










      • $begingroup$
        @ChrisMillet You're welcome! I hope it makes sense.
        $endgroup$
        – layman
        Feb 4 '15 at 20:33
















      • 1




        $begingroup$
        Of the four answers so far, this is the only one that answers the question of “how to go about solving this problem”.
        $endgroup$
        – MJD
        Feb 4 '15 at 17:48










      • $begingroup$
        @user46944 thank you so much for explaining this. He said that this one would make us think for a while. I assume he said this because we haven't fully touched upon this yet. Thanks for the indepth and comprehensive answer
        $endgroup$
        – Chris Millett
        Feb 4 '15 at 18:59










      • $begingroup$
        @ChrisMillet You're welcome! I hope it makes sense.
        $endgroup$
        – layman
        Feb 4 '15 at 20:33










      1




      1




      $begingroup$
      Of the four answers so far, this is the only one that answers the question of “how to go about solving this problem”.
      $endgroup$
      – MJD
      Feb 4 '15 at 17:48




      $begingroup$
      Of the four answers so far, this is the only one that answers the question of “how to go about solving this problem”.
      $endgroup$
      – MJD
      Feb 4 '15 at 17:48












      $begingroup$
      @user46944 thank you so much for explaining this. He said that this one would make us think for a while. I assume he said this because we haven't fully touched upon this yet. Thanks for the indepth and comprehensive answer
      $endgroup$
      – Chris Millett
      Feb 4 '15 at 18:59




      $begingroup$
      @user46944 thank you so much for explaining this. He said that this one would make us think for a while. I assume he said this because we haven't fully touched upon this yet. Thanks for the indepth and comprehensive answer
      $endgroup$
      – Chris Millett
      Feb 4 '15 at 18:59












      $begingroup$
      @ChrisMillet You're welcome! I hope it makes sense.
      $endgroup$
      – layman
      Feb 4 '15 at 20:33






      $begingroup$
      @ChrisMillet You're welcome! I hope it makes sense.
      $endgroup$
      – layman
      Feb 4 '15 at 20:33













      2












      $begingroup$

      Think about...
      $$
      bigcup_{n=1}^{infty}left(0+frac{1}{n},1right)
      $$






      share|cite|improve this answer









      $endgroup$


















        2












        $begingroup$

        Think about...
        $$
        bigcup_{n=1}^{infty}left(0+frac{1}{n},1right)
        $$






        share|cite|improve this answer









        $endgroup$
















          2












          2








          2





          $begingroup$

          Think about...
          $$
          bigcup_{n=1}^{infty}left(0+frac{1}{n},1right)
          $$






          share|cite|improve this answer









          $endgroup$



          Think about...
          $$
          bigcup_{n=1}^{infty}left(0+frac{1}{n},1right)
          $$







          share|cite|improve this answer












          share|cite|improve this answer



          share|cite|improve this answer










          answered Feb 4 '15 at 17:07









          parsiadparsiad

          18.5k32453




          18.5k32453























              2












              $begingroup$

              You can take the open cover ${(0+frac{1}{n}, 1-frac{1}{n})mid n>2} $. It has no finite subcover because if it does simply take the largest $n$ and $frac{1}{2n}$ is not included.






              share|cite|improve this answer









              $endgroup$













              • $begingroup$
                I never thought of it like that. I assume it's because we havent fully finished the topic yet and I wasn't sure whether or not to keep reading thanks!
                $endgroup$
                – Chris Millett
                Feb 4 '15 at 19:03
















              2












              $begingroup$

              You can take the open cover ${(0+frac{1}{n}, 1-frac{1}{n})mid n>2} $. It has no finite subcover because if it does simply take the largest $n$ and $frac{1}{2n}$ is not included.






              share|cite|improve this answer









              $endgroup$













              • $begingroup$
                I never thought of it like that. I assume it's because we havent fully finished the topic yet and I wasn't sure whether or not to keep reading thanks!
                $endgroup$
                – Chris Millett
                Feb 4 '15 at 19:03














              2












              2








              2





              $begingroup$

              You can take the open cover ${(0+frac{1}{n}, 1-frac{1}{n})mid n>2} $. It has no finite subcover because if it does simply take the largest $n$ and $frac{1}{2n}$ is not included.






              share|cite|improve this answer









              $endgroup$



              You can take the open cover ${(0+frac{1}{n}, 1-frac{1}{n})mid n>2} $. It has no finite subcover because if it does simply take the largest $n$ and $frac{1}{2n}$ is not included.







              share|cite|improve this answer












              share|cite|improve this answer



              share|cite|improve this answer










              answered Feb 4 '15 at 17:09









              QidiQidi

              1,110712




              1,110712












              • $begingroup$
                I never thought of it like that. I assume it's because we havent fully finished the topic yet and I wasn't sure whether or not to keep reading thanks!
                $endgroup$
                – Chris Millett
                Feb 4 '15 at 19:03


















              • $begingroup$
                I never thought of it like that. I assume it's because we havent fully finished the topic yet and I wasn't sure whether or not to keep reading thanks!
                $endgroup$
                – Chris Millett
                Feb 4 '15 at 19:03
















              $begingroup$
              I never thought of it like that. I assume it's because we havent fully finished the topic yet and I wasn't sure whether or not to keep reading thanks!
              $endgroup$
              – Chris Millett
              Feb 4 '15 at 19:03




              $begingroup$
              I never thought of it like that. I assume it's because we havent fully finished the topic yet and I wasn't sure whether or not to keep reading thanks!
              $endgroup$
              – Chris Millett
              Feb 4 '15 at 19:03











              2












              $begingroup$

              Since you have the real analysis tag, I'm assuming you are using the standard topology on $Bbb{R}$, so open sets look like intervals of the form $(a,b)$. An open cover of $(0,1)$ could look something like $$bigcup_{i=1}^infty left(0,1-frac{1}{i} right)$$ But you need to verify this is an open cover. That is, show $$(0,1) subseteq bigcup_{i=1}^infty left(0,1-frac{1}{i} right)$$ and further show that if $K subset Bbb{N}$ is any finite subset of $Bbb{N}$ that $$(0,1) notsubseteq bigcup_{iin K} left(0,1-frac{1}{i} right)$$ which demonstrates that the open cover has no finite subcover.






              share|cite|improve this answer









              $endgroup$













              • $begingroup$
                gotcha this makes sense too. Thanks a bunch
                $endgroup$
                – Chris Millett
                Feb 4 '15 at 19:00
















              2












              $begingroup$

              Since you have the real analysis tag, I'm assuming you are using the standard topology on $Bbb{R}$, so open sets look like intervals of the form $(a,b)$. An open cover of $(0,1)$ could look something like $$bigcup_{i=1}^infty left(0,1-frac{1}{i} right)$$ But you need to verify this is an open cover. That is, show $$(0,1) subseteq bigcup_{i=1}^infty left(0,1-frac{1}{i} right)$$ and further show that if $K subset Bbb{N}$ is any finite subset of $Bbb{N}$ that $$(0,1) notsubseteq bigcup_{iin K} left(0,1-frac{1}{i} right)$$ which demonstrates that the open cover has no finite subcover.






              share|cite|improve this answer









              $endgroup$













              • $begingroup$
                gotcha this makes sense too. Thanks a bunch
                $endgroup$
                – Chris Millett
                Feb 4 '15 at 19:00














              2












              2








              2





              $begingroup$

              Since you have the real analysis tag, I'm assuming you are using the standard topology on $Bbb{R}$, so open sets look like intervals of the form $(a,b)$. An open cover of $(0,1)$ could look something like $$bigcup_{i=1}^infty left(0,1-frac{1}{i} right)$$ But you need to verify this is an open cover. That is, show $$(0,1) subseteq bigcup_{i=1}^infty left(0,1-frac{1}{i} right)$$ and further show that if $K subset Bbb{N}$ is any finite subset of $Bbb{N}$ that $$(0,1) notsubseteq bigcup_{iin K} left(0,1-frac{1}{i} right)$$ which demonstrates that the open cover has no finite subcover.






              share|cite|improve this answer









              $endgroup$



              Since you have the real analysis tag, I'm assuming you are using the standard topology on $Bbb{R}$, so open sets look like intervals of the form $(a,b)$. An open cover of $(0,1)$ could look something like $$bigcup_{i=1}^infty left(0,1-frac{1}{i} right)$$ But you need to verify this is an open cover. That is, show $$(0,1) subseteq bigcup_{i=1}^infty left(0,1-frac{1}{i} right)$$ and further show that if $K subset Bbb{N}$ is any finite subset of $Bbb{N}$ that $$(0,1) notsubseteq bigcup_{iin K} left(0,1-frac{1}{i} right)$$ which demonstrates that the open cover has no finite subcover.







              share|cite|improve this answer












              share|cite|improve this answer



              share|cite|improve this answer










              answered Feb 4 '15 at 17:15









              graydadgraydad

              12.8k61933




              12.8k61933












              • $begingroup$
                gotcha this makes sense too. Thanks a bunch
                $endgroup$
                – Chris Millett
                Feb 4 '15 at 19:00


















              • $begingroup$
                gotcha this makes sense too. Thanks a bunch
                $endgroup$
                – Chris Millett
                Feb 4 '15 at 19:00
















              $begingroup$
              gotcha this makes sense too. Thanks a bunch
              $endgroup$
              – Chris Millett
              Feb 4 '15 at 19:00




              $begingroup$
              gotcha this makes sense too. Thanks a bunch
              $endgroup$
              – Chris Millett
              Feb 4 '15 at 19:00











              0












              $begingroup$

              In case you are still interested. Abbot's has this problem as Example 3.3.7. For each point $xin(0,1)$ consider the open interval $0_x = (x/2,1)$. The infinite union of $O_x$'s forms an open cover of $(0,1)$. Take any finite subset of such $O_x$, and set $x'=operatorname{min}(x_1,x_2,dots,x_n)$. You can still find a number $0<yleq x'/2$ not in any subfinite union.






              share|cite|improve this answer









              $endgroup$


















                0












                $begingroup$

                In case you are still interested. Abbot's has this problem as Example 3.3.7. For each point $xin(0,1)$ consider the open interval $0_x = (x/2,1)$. The infinite union of $O_x$'s forms an open cover of $(0,1)$. Take any finite subset of such $O_x$, and set $x'=operatorname{min}(x_1,x_2,dots,x_n)$. You can still find a number $0<yleq x'/2$ not in any subfinite union.






                share|cite|improve this answer









                $endgroup$
















                  0












                  0








                  0





                  $begingroup$

                  In case you are still interested. Abbot's has this problem as Example 3.3.7. For each point $xin(0,1)$ consider the open interval $0_x = (x/2,1)$. The infinite union of $O_x$'s forms an open cover of $(0,1)$. Take any finite subset of such $O_x$, and set $x'=operatorname{min}(x_1,x_2,dots,x_n)$. You can still find a number $0<yleq x'/2$ not in any subfinite union.






                  share|cite|improve this answer









                  $endgroup$



                  In case you are still interested. Abbot's has this problem as Example 3.3.7. For each point $xin(0,1)$ consider the open interval $0_x = (x/2,1)$. The infinite union of $O_x$'s forms an open cover of $(0,1)$. Take any finite subset of such $O_x$, and set $x'=operatorname{min}(x_1,x_2,dots,x_n)$. You can still find a number $0<yleq x'/2$ not in any subfinite union.







                  share|cite|improve this answer












                  share|cite|improve this answer



                  share|cite|improve this answer










                  answered Mar 14 at 19:39









                  user2820579user2820579

                  782417




                  782417






























                      draft saved

                      draft discarded




















































                      Thanks for contributing an answer to Mathematics Stack Exchange!


                      • Please be sure to answer the question. Provide details and share your research!

                      But avoid



                      • Asking for help, clarification, or responding to other answers.

                      • Making statements based on opinion; back them up with references or personal experience.


                      Use MathJax to format equations. MathJax reference.


                      To learn more, see our tips on writing great answers.




                      draft saved


                      draft discarded














                      StackExchange.ready(
                      function () {
                      StackExchange.openid.initPostLogin('.new-post-login', 'https%3a%2f%2fmath.stackexchange.com%2fquestions%2f1133580%2fan-example-of-an-infinite-open-cover-of-the-interval-0-1-that-has-no-finite-su%23new-answer', 'question_page');
                      }
                      );

                      Post as a guest















                      Required, but never shown





















































                      Required, but never shown














                      Required, but never shown












                      Required, but never shown







                      Required, but never shown

































                      Required, but never shown














                      Required, but never shown












                      Required, but never shown







                      Required, but never shown







                      Popular posts from this blog

                      Magento 2 - Add success message with knockout Planned maintenance scheduled April 23, 2019 at 23:30 UTC (7:30pm US/Eastern) Announcing the arrival of Valued Associate #679: Cesar Manara Unicorn Meta Zoo #1: Why another podcast?Success / Error message on ajax request$.widget is not a function when loading a homepage after add custom jQuery on custom themeHow can bind jQuery to current document in Magento 2 When template load by ajaxRedirect page using plugin in Magento 2Magento 2 - Update quantity and totals of cart page without page reload?Magento 2: Quote data not loaded on knockout checkoutMagento 2 : I need to change add to cart success message after adding product into cart through pluginMagento 2.2.5 How to add additional products to cart from new checkout step?Magento 2 Add error/success message with knockoutCan't validate Post Code on checkout page

                      Fil:Tokke komm.svg

                      Where did Arya get these scars? Unicorn Meta Zoo #1: Why another podcast? Announcing the arrival of Valued Associate #679: Cesar Manara Favourite questions and answers from the 1st quarter of 2019Why did Arya refuse to end it?Has the pronunciation of Arya Stark's name changed?Has Arya forgiven people?Why did Arya Stark lose her vision?Why can Arya still use the faces?Has the Narrow Sea become narrower?Does Arya Stark know how to make poisons outside of the House of Black and White?Why did Nymeria leave Arya?Why did Arya not kill the Lannister soldiers she encountered in the Riverlands?What is the current canonical age of Sansa, Bran and Arya Stark?